Clarification on alternative expression of sumHow to find the general solution of $(1+x^2)y''+2xy'-2y=0$. How...

How were servants to the Kaiser of Imperial Germany treated and where may I find more information on them

What is the smallest number n> 5 so that 5 ^ n ends with "3125"?

Why would five hundred and five be same as one?

How to test the sharpness of a knife?

Why can't the Brexit deadlock in the UK parliament be solved with a plurality vote?

Unable to disable Microsoft Store in domain environment

How to preserve electronics (computers, iPads and phones) for hundreds of years

What (the heck) is a Super Worm Equinox Moon?

Can I run 125kHz RF circuit on a breadboard?

How to make money from a browser who sees 5 seconds into the future of any web page?

El Dorado Word Puzzle II: Videogame Edition

Can I say "fingers" when referring to toes?

If the only attacker is removed from combat, is a creature still counted as having attacked this turn?

Pre-Employment Background Check With Consent For Future Checks

Check if object is null and return null

Do I have to take mana from my deck or hand when tapping a dual land?

Is there a way to play vibrato on the piano?

Why didn’t Eve recognize the little cockroach as a living organism?

Anime with legendary swords made from talismans and a man who could change them with a shattered body

When and why was runway 07/25 at Kai Tak removed?

Why the "ls" command is showing the permissions of files in a FAT32 partition?

How do I tell my boss that I'm quitting in 15 days (a colleague left this week)

Why is participating in the European Parliamentary elections used as a threat?

Overlapping circles covering polygon



Clarification on alternative expression of sum


How to find the general solution of $(1+x^2)y''+2xy'-2y=0$. How to express by means of elementary functions?Multiplying two summations together exactly.Expressing $zinmathbb{C}[[w]]$ as a power series in $yinmathbb{C}[[z]]$.Expression by Power Series Expansion and Binomial ExpansionChanging summation in a power seriesExplicit expression of a given power seriesPairwise sum and divideCauchy -Schwarz for double summationI know my series converges; how do I get a closed-form expression for it?Evaluating Sum of $dfrac{i}{(-x)^i}$













1












$begingroup$


I am not familiar with alternative expression of sum shown below,



$ d_k=sum_{i+j+l=k}a_ib_jc_l $



How it does work?



for $k = 4 $ then,



$d_{4} = sum_{i+j+l=4}a_ib_jc_l = ...$



How do I express it in standard summation format namely $sum_{n=0}^{infty}$
This is from the formula to calculate the product of three summations (power series) or $sum_{i=0}^{infty} a_i x^i cdot sum_{j=0}^{infty} b_j x^j cdot sum_{l=0}^{infty} c_l x^l = sum_{k=0}^{infty} d_kx^k$










share|cite|improve this question











$endgroup$












  • $begingroup$
    Can't you solve $i+j+l=4$ ?
    $endgroup$
    – Yves Daoust
    Mar 12 at 22:07
















1












$begingroup$


I am not familiar with alternative expression of sum shown below,



$ d_k=sum_{i+j+l=k}a_ib_jc_l $



How it does work?



for $k = 4 $ then,



$d_{4} = sum_{i+j+l=4}a_ib_jc_l = ...$



How do I express it in standard summation format namely $sum_{n=0}^{infty}$
This is from the formula to calculate the product of three summations (power series) or $sum_{i=0}^{infty} a_i x^i cdot sum_{j=0}^{infty} b_j x^j cdot sum_{l=0}^{infty} c_l x^l = sum_{k=0}^{infty} d_kx^k$










share|cite|improve this question











$endgroup$












  • $begingroup$
    Can't you solve $i+j+l=4$ ?
    $endgroup$
    – Yves Daoust
    Mar 12 at 22:07














1












1








1


0



$begingroup$


I am not familiar with alternative expression of sum shown below,



$ d_k=sum_{i+j+l=k}a_ib_jc_l $



How it does work?



for $k = 4 $ then,



$d_{4} = sum_{i+j+l=4}a_ib_jc_l = ...$



How do I express it in standard summation format namely $sum_{n=0}^{infty}$
This is from the formula to calculate the product of three summations (power series) or $sum_{i=0}^{infty} a_i x^i cdot sum_{j=0}^{infty} b_j x^j cdot sum_{l=0}^{infty} c_l x^l = sum_{k=0}^{infty} d_kx^k$










share|cite|improve this question











$endgroup$




I am not familiar with alternative expression of sum shown below,



$ d_k=sum_{i+j+l=k}a_ib_jc_l $



How it does work?



for $k = 4 $ then,



$d_{4} = sum_{i+j+l=4}a_ib_jc_l = ...$



How do I express it in standard summation format namely $sum_{n=0}^{infty}$
This is from the formula to calculate the product of three summations (power series) or $sum_{i=0}^{infty} a_i x^i cdot sum_{j=0}^{infty} b_j x^j cdot sum_{l=0}^{infty} c_l x^l = sum_{k=0}^{infty} d_kx^k$







summation power-series






share|cite|improve this question















share|cite|improve this question













share|cite|improve this question




share|cite|improve this question








edited Mar 13 at 15:04







Aschoolar

















asked Mar 12 at 22:03









AschoolarAschoolar

2131210




2131210












  • $begingroup$
    Can't you solve $i+j+l=4$ ?
    $endgroup$
    – Yves Daoust
    Mar 12 at 22:07


















  • $begingroup$
    Can't you solve $i+j+l=4$ ?
    $endgroup$
    – Yves Daoust
    Mar 12 at 22:07
















$begingroup$
Can't you solve $i+j+l=4$ ?
$endgroup$
– Yves Daoust
Mar 12 at 22:07




$begingroup$
Can't you solve $i+j+l=4$ ?
$endgroup$
– Yves Daoust
Mar 12 at 22:07










3 Answers
3






active

oldest

votes


















1












$begingroup$

First, it is assumed that the indices are non-negative.



Second step, it means that the sum is to be taken for the triples $(i,j,l)$ for which $i+j+l=k |0 le i,j,l$. These are the "weak" compositions of $k$ into three parts



So for e.g. $k=4$ you have to sum $a_0b_0c_4+cdots+a_0b_1c_3+cdots$






share|cite|improve this answer









$endgroup$





















    1












    $begingroup$

    You can get
    explicit indices like this
    (assuming that
    the lower index of summation
    is $0$):



    $begin{array}\
    d_k
    &=sum_{i+j+l=k}a_ib_jc_l\
    &=sum_{i=0}^{k}
    a_i sum_{j=0}^{k-i}b_jc_{k-i-j}\
    end{array}
    $



    If the summation
    is over an inequality,
    you get one more level
    of summation:



    $begin{array}\
    e_k
    &=sum_{0 le i+j+l le k}a_ib_jc_l\
    &=sum_{i=0}^{k}
    a_i sum_{j=0}^{k-i}b_jsum_{l=0}^{k-i-j}c_{l}\
    end{array}
    $



    Work out for yourself
    what these are
    if the lower index of summation
    is $1$ instead of $0$.






    share|cite|improve this answer









    $endgroup$





















      0












      $begingroup$

      The representation of $d_k$ as triple sum $sum_{i+j+l=k}a_ib_jc_l$ can be derived by applying Cauchy Series multiplication twice.




      We obtain
      begin{align*}
      left(sum_{i=0}^inftyright.&left. a_i x^iright)left(sum_{j=0}^infty b_j x^jright)left(sum_{j=0}^infty b_j x^jright)\
      &=left(sum_{n=0}^inftyleft(sum_{{i+j=n}atop{i,jgeq 0}}a_ib_jright)x^nright)left(sum_{j=0}^infty b_j x^jright)tag{1}\
      &=sum_{k=0}^inftyleft(sum_{{n+l=k}atop{n,lgeq 0}}left(sum_{{i+j=n}atop{i,jgeq 0}}right)c_lright)x^k\
      &=sum_{k=0}^inftyleft(color{blue}{sum_{{i+j+l=k}atop{i,j,l}}a_ib_jc_l}right)x^ktag{2}\
      end{align*}




      From (1) we also obtain




      begin{align*}
      left(sum_{n=0}^inftyright.&left.left(sum_{{i+j=n}atop{i,jgeq 0}}a_ib_jright)x^nright)left(sum_{j=0}^infty b_j x^jright)\
      &=left(sum_{n=0}^inftyleft(sum_{i=0}^na_ib_{n-i}right)x^nright)left(sum_{l=0}^infty c_lx^lright)\
      &=sum_{k=0}^inftyleft(sum_{{n+l=k}atop{n,lgeq 0}}left(sum_{i=0}^na_ib_{n-i}right)c_lright)x^l\
      &=sum_{k=0}^inftyleft(color{blue}{sum_{n=0}^ksum_{i=0}^na_ib_{n-i}c_{k-n}}right)x^ktag{3}
      end{align*}

      Comparing coefficients of equal powers of $x$ in (2) and (3) shows equality of the sums.







      share|cite|improve this answer











      $endgroup$













        Your Answer





        StackExchange.ifUsing("editor", function () {
        return StackExchange.using("mathjaxEditing", function () {
        StackExchange.MarkdownEditor.creationCallbacks.add(function (editor, postfix) {
        StackExchange.mathjaxEditing.prepareWmdForMathJax(editor, postfix, [["$", "$"], ["\\(","\\)"]]);
        });
        });
        }, "mathjax-editing");

        StackExchange.ready(function() {
        var channelOptions = {
        tags: "".split(" "),
        id: "69"
        };
        initTagRenderer("".split(" "), "".split(" "), channelOptions);

        StackExchange.using("externalEditor", function() {
        // Have to fire editor after snippets, if snippets enabled
        if (StackExchange.settings.snippets.snippetsEnabled) {
        StackExchange.using("snippets", function() {
        createEditor();
        });
        }
        else {
        createEditor();
        }
        });

        function createEditor() {
        StackExchange.prepareEditor({
        heartbeatType: 'answer',
        autoActivateHeartbeat: false,
        convertImagesToLinks: true,
        noModals: true,
        showLowRepImageUploadWarning: true,
        reputationToPostImages: 10,
        bindNavPrevention: true,
        postfix: "",
        imageUploader: {
        brandingHtml: "Powered by u003ca class="icon-imgur-white" href="https://imgur.com/"u003eu003c/au003e",
        contentPolicyHtml: "User contributions licensed under u003ca href="https://creativecommons.org/licenses/by-sa/3.0/"u003ecc by-sa 3.0 with attribution requiredu003c/au003e u003ca href="https://stackoverflow.com/legal/content-policy"u003e(content policy)u003c/au003e",
        allowUrls: true
        },
        noCode: true, onDemand: true,
        discardSelector: ".discard-answer"
        ,immediatelyShowMarkdownHelp:true
        });


        }
        });














        draft saved

        draft discarded


















        StackExchange.ready(
        function () {
        StackExchange.openid.initPostLogin('.new-post-login', 'https%3a%2f%2fmath.stackexchange.com%2fquestions%2f3145761%2fclarification-on-alternative-expression-of-sum%23new-answer', 'question_page');
        }
        );

        Post as a guest















        Required, but never shown

























        3 Answers
        3






        active

        oldest

        votes








        3 Answers
        3






        active

        oldest

        votes









        active

        oldest

        votes






        active

        oldest

        votes









        1












        $begingroup$

        First, it is assumed that the indices are non-negative.



        Second step, it means that the sum is to be taken for the triples $(i,j,l)$ for which $i+j+l=k |0 le i,j,l$. These are the "weak" compositions of $k$ into three parts



        So for e.g. $k=4$ you have to sum $a_0b_0c_4+cdots+a_0b_1c_3+cdots$






        share|cite|improve this answer









        $endgroup$


















          1












          $begingroup$

          First, it is assumed that the indices are non-negative.



          Second step, it means that the sum is to be taken for the triples $(i,j,l)$ for which $i+j+l=k |0 le i,j,l$. These are the "weak" compositions of $k$ into three parts



          So for e.g. $k=4$ you have to sum $a_0b_0c_4+cdots+a_0b_1c_3+cdots$






          share|cite|improve this answer









          $endgroup$
















            1












            1








            1





            $begingroup$

            First, it is assumed that the indices are non-negative.



            Second step, it means that the sum is to be taken for the triples $(i,j,l)$ for which $i+j+l=k |0 le i,j,l$. These are the "weak" compositions of $k$ into three parts



            So for e.g. $k=4$ you have to sum $a_0b_0c_4+cdots+a_0b_1c_3+cdots$






            share|cite|improve this answer









            $endgroup$



            First, it is assumed that the indices are non-negative.



            Second step, it means that the sum is to be taken for the triples $(i,j,l)$ for which $i+j+l=k |0 le i,j,l$. These are the "weak" compositions of $k$ into three parts



            So for e.g. $k=4$ you have to sum $a_0b_0c_4+cdots+a_0b_1c_3+cdots$







            share|cite|improve this answer












            share|cite|improve this answer



            share|cite|improve this answer










            answered Mar 12 at 22:18









            G CabG Cab

            20.4k31341




            20.4k31341























                1












                $begingroup$

                You can get
                explicit indices like this
                (assuming that
                the lower index of summation
                is $0$):



                $begin{array}\
                d_k
                &=sum_{i+j+l=k}a_ib_jc_l\
                &=sum_{i=0}^{k}
                a_i sum_{j=0}^{k-i}b_jc_{k-i-j}\
                end{array}
                $



                If the summation
                is over an inequality,
                you get one more level
                of summation:



                $begin{array}\
                e_k
                &=sum_{0 le i+j+l le k}a_ib_jc_l\
                &=sum_{i=0}^{k}
                a_i sum_{j=0}^{k-i}b_jsum_{l=0}^{k-i-j}c_{l}\
                end{array}
                $



                Work out for yourself
                what these are
                if the lower index of summation
                is $1$ instead of $0$.






                share|cite|improve this answer









                $endgroup$


















                  1












                  $begingroup$

                  You can get
                  explicit indices like this
                  (assuming that
                  the lower index of summation
                  is $0$):



                  $begin{array}\
                  d_k
                  &=sum_{i+j+l=k}a_ib_jc_l\
                  &=sum_{i=0}^{k}
                  a_i sum_{j=0}^{k-i}b_jc_{k-i-j}\
                  end{array}
                  $



                  If the summation
                  is over an inequality,
                  you get one more level
                  of summation:



                  $begin{array}\
                  e_k
                  &=sum_{0 le i+j+l le k}a_ib_jc_l\
                  &=sum_{i=0}^{k}
                  a_i sum_{j=0}^{k-i}b_jsum_{l=0}^{k-i-j}c_{l}\
                  end{array}
                  $



                  Work out for yourself
                  what these are
                  if the lower index of summation
                  is $1$ instead of $0$.






                  share|cite|improve this answer









                  $endgroup$
















                    1












                    1








                    1





                    $begingroup$

                    You can get
                    explicit indices like this
                    (assuming that
                    the lower index of summation
                    is $0$):



                    $begin{array}\
                    d_k
                    &=sum_{i+j+l=k}a_ib_jc_l\
                    &=sum_{i=0}^{k}
                    a_i sum_{j=0}^{k-i}b_jc_{k-i-j}\
                    end{array}
                    $



                    If the summation
                    is over an inequality,
                    you get one more level
                    of summation:



                    $begin{array}\
                    e_k
                    &=sum_{0 le i+j+l le k}a_ib_jc_l\
                    &=sum_{i=0}^{k}
                    a_i sum_{j=0}^{k-i}b_jsum_{l=0}^{k-i-j}c_{l}\
                    end{array}
                    $



                    Work out for yourself
                    what these are
                    if the lower index of summation
                    is $1$ instead of $0$.






                    share|cite|improve this answer









                    $endgroup$



                    You can get
                    explicit indices like this
                    (assuming that
                    the lower index of summation
                    is $0$):



                    $begin{array}\
                    d_k
                    &=sum_{i+j+l=k}a_ib_jc_l\
                    &=sum_{i=0}^{k}
                    a_i sum_{j=0}^{k-i}b_jc_{k-i-j}\
                    end{array}
                    $



                    If the summation
                    is over an inequality,
                    you get one more level
                    of summation:



                    $begin{array}\
                    e_k
                    &=sum_{0 le i+j+l le k}a_ib_jc_l\
                    &=sum_{i=0}^{k}
                    a_i sum_{j=0}^{k-i}b_jsum_{l=0}^{k-i-j}c_{l}\
                    end{array}
                    $



                    Work out for yourself
                    what these are
                    if the lower index of summation
                    is $1$ instead of $0$.







                    share|cite|improve this answer












                    share|cite|improve this answer



                    share|cite|improve this answer










                    answered Mar 12 at 22:56









                    marty cohenmarty cohen

                    74.4k549129




                    74.4k549129























                        0












                        $begingroup$

                        The representation of $d_k$ as triple sum $sum_{i+j+l=k}a_ib_jc_l$ can be derived by applying Cauchy Series multiplication twice.




                        We obtain
                        begin{align*}
                        left(sum_{i=0}^inftyright.&left. a_i x^iright)left(sum_{j=0}^infty b_j x^jright)left(sum_{j=0}^infty b_j x^jright)\
                        &=left(sum_{n=0}^inftyleft(sum_{{i+j=n}atop{i,jgeq 0}}a_ib_jright)x^nright)left(sum_{j=0}^infty b_j x^jright)tag{1}\
                        &=sum_{k=0}^inftyleft(sum_{{n+l=k}atop{n,lgeq 0}}left(sum_{{i+j=n}atop{i,jgeq 0}}right)c_lright)x^k\
                        &=sum_{k=0}^inftyleft(color{blue}{sum_{{i+j+l=k}atop{i,j,l}}a_ib_jc_l}right)x^ktag{2}\
                        end{align*}




                        From (1) we also obtain




                        begin{align*}
                        left(sum_{n=0}^inftyright.&left.left(sum_{{i+j=n}atop{i,jgeq 0}}a_ib_jright)x^nright)left(sum_{j=0}^infty b_j x^jright)\
                        &=left(sum_{n=0}^inftyleft(sum_{i=0}^na_ib_{n-i}right)x^nright)left(sum_{l=0}^infty c_lx^lright)\
                        &=sum_{k=0}^inftyleft(sum_{{n+l=k}atop{n,lgeq 0}}left(sum_{i=0}^na_ib_{n-i}right)c_lright)x^l\
                        &=sum_{k=0}^inftyleft(color{blue}{sum_{n=0}^ksum_{i=0}^na_ib_{n-i}c_{k-n}}right)x^ktag{3}
                        end{align*}

                        Comparing coefficients of equal powers of $x$ in (2) and (3) shows equality of the sums.







                        share|cite|improve this answer











                        $endgroup$


















                          0












                          $begingroup$

                          The representation of $d_k$ as triple sum $sum_{i+j+l=k}a_ib_jc_l$ can be derived by applying Cauchy Series multiplication twice.




                          We obtain
                          begin{align*}
                          left(sum_{i=0}^inftyright.&left. a_i x^iright)left(sum_{j=0}^infty b_j x^jright)left(sum_{j=0}^infty b_j x^jright)\
                          &=left(sum_{n=0}^inftyleft(sum_{{i+j=n}atop{i,jgeq 0}}a_ib_jright)x^nright)left(sum_{j=0}^infty b_j x^jright)tag{1}\
                          &=sum_{k=0}^inftyleft(sum_{{n+l=k}atop{n,lgeq 0}}left(sum_{{i+j=n}atop{i,jgeq 0}}right)c_lright)x^k\
                          &=sum_{k=0}^inftyleft(color{blue}{sum_{{i+j+l=k}atop{i,j,l}}a_ib_jc_l}right)x^ktag{2}\
                          end{align*}




                          From (1) we also obtain




                          begin{align*}
                          left(sum_{n=0}^inftyright.&left.left(sum_{{i+j=n}atop{i,jgeq 0}}a_ib_jright)x^nright)left(sum_{j=0}^infty b_j x^jright)\
                          &=left(sum_{n=0}^inftyleft(sum_{i=0}^na_ib_{n-i}right)x^nright)left(sum_{l=0}^infty c_lx^lright)\
                          &=sum_{k=0}^inftyleft(sum_{{n+l=k}atop{n,lgeq 0}}left(sum_{i=0}^na_ib_{n-i}right)c_lright)x^l\
                          &=sum_{k=0}^inftyleft(color{blue}{sum_{n=0}^ksum_{i=0}^na_ib_{n-i}c_{k-n}}right)x^ktag{3}
                          end{align*}

                          Comparing coefficients of equal powers of $x$ in (2) and (3) shows equality of the sums.







                          share|cite|improve this answer











                          $endgroup$
















                            0












                            0








                            0





                            $begingroup$

                            The representation of $d_k$ as triple sum $sum_{i+j+l=k}a_ib_jc_l$ can be derived by applying Cauchy Series multiplication twice.




                            We obtain
                            begin{align*}
                            left(sum_{i=0}^inftyright.&left. a_i x^iright)left(sum_{j=0}^infty b_j x^jright)left(sum_{j=0}^infty b_j x^jright)\
                            &=left(sum_{n=0}^inftyleft(sum_{{i+j=n}atop{i,jgeq 0}}a_ib_jright)x^nright)left(sum_{j=0}^infty b_j x^jright)tag{1}\
                            &=sum_{k=0}^inftyleft(sum_{{n+l=k}atop{n,lgeq 0}}left(sum_{{i+j=n}atop{i,jgeq 0}}right)c_lright)x^k\
                            &=sum_{k=0}^inftyleft(color{blue}{sum_{{i+j+l=k}atop{i,j,l}}a_ib_jc_l}right)x^ktag{2}\
                            end{align*}




                            From (1) we also obtain




                            begin{align*}
                            left(sum_{n=0}^inftyright.&left.left(sum_{{i+j=n}atop{i,jgeq 0}}a_ib_jright)x^nright)left(sum_{j=0}^infty b_j x^jright)\
                            &=left(sum_{n=0}^inftyleft(sum_{i=0}^na_ib_{n-i}right)x^nright)left(sum_{l=0}^infty c_lx^lright)\
                            &=sum_{k=0}^inftyleft(sum_{{n+l=k}atop{n,lgeq 0}}left(sum_{i=0}^na_ib_{n-i}right)c_lright)x^l\
                            &=sum_{k=0}^inftyleft(color{blue}{sum_{n=0}^ksum_{i=0}^na_ib_{n-i}c_{k-n}}right)x^ktag{3}
                            end{align*}

                            Comparing coefficients of equal powers of $x$ in (2) and (3) shows equality of the sums.







                            share|cite|improve this answer











                            $endgroup$



                            The representation of $d_k$ as triple sum $sum_{i+j+l=k}a_ib_jc_l$ can be derived by applying Cauchy Series multiplication twice.




                            We obtain
                            begin{align*}
                            left(sum_{i=0}^inftyright.&left. a_i x^iright)left(sum_{j=0}^infty b_j x^jright)left(sum_{j=0}^infty b_j x^jright)\
                            &=left(sum_{n=0}^inftyleft(sum_{{i+j=n}atop{i,jgeq 0}}a_ib_jright)x^nright)left(sum_{j=0}^infty b_j x^jright)tag{1}\
                            &=sum_{k=0}^inftyleft(sum_{{n+l=k}atop{n,lgeq 0}}left(sum_{{i+j=n}atop{i,jgeq 0}}right)c_lright)x^k\
                            &=sum_{k=0}^inftyleft(color{blue}{sum_{{i+j+l=k}atop{i,j,l}}a_ib_jc_l}right)x^ktag{2}\
                            end{align*}




                            From (1) we also obtain




                            begin{align*}
                            left(sum_{n=0}^inftyright.&left.left(sum_{{i+j=n}atop{i,jgeq 0}}a_ib_jright)x^nright)left(sum_{j=0}^infty b_j x^jright)\
                            &=left(sum_{n=0}^inftyleft(sum_{i=0}^na_ib_{n-i}right)x^nright)left(sum_{l=0}^infty c_lx^lright)\
                            &=sum_{k=0}^inftyleft(sum_{{n+l=k}atop{n,lgeq 0}}left(sum_{i=0}^na_ib_{n-i}right)c_lright)x^l\
                            &=sum_{k=0}^inftyleft(color{blue}{sum_{n=0}^ksum_{i=0}^na_ib_{n-i}c_{k-n}}right)x^ktag{3}
                            end{align*}

                            Comparing coefficients of equal powers of $x$ in (2) and (3) shows equality of the sums.








                            share|cite|improve this answer














                            share|cite|improve this answer



                            share|cite|improve this answer








                            edited Mar 15 at 7:08

























                            answered Mar 14 at 22:08









                            Markus ScheuerMarkus Scheuer

                            62.7k460150




                            62.7k460150






























                                draft saved

                                draft discarded




















































                                Thanks for contributing an answer to Mathematics Stack Exchange!


                                • Please be sure to answer the question. Provide details and share your research!

                                But avoid



                                • Asking for help, clarification, or responding to other answers.

                                • Making statements based on opinion; back them up with references or personal experience.


                                Use MathJax to format equations. MathJax reference.


                                To learn more, see our tips on writing great answers.




                                draft saved


                                draft discarded














                                StackExchange.ready(
                                function () {
                                StackExchange.openid.initPostLogin('.new-post-login', 'https%3a%2f%2fmath.stackexchange.com%2fquestions%2f3145761%2fclarification-on-alternative-expression-of-sum%23new-answer', 'question_page');
                                }
                                );

                                Post as a guest















                                Required, but never shown





















































                                Required, but never shown














                                Required, but never shown












                                Required, but never shown







                                Required, but never shown

































                                Required, but never shown














                                Required, but never shown












                                Required, but never shown







                                Required, but never shown







                                Popular posts from this blog

                                Magento 2 - Add success message with knockout Planned maintenance scheduled April 23, 2019 at 23:30 UTC (7:30pm US/Eastern) Announcing the arrival of Valued Associate #679: Cesar Manara Unicorn Meta Zoo #1: Why another podcast?Success / Error message on ajax request$.widget is not a function when loading a homepage after add custom jQuery on custom themeHow can bind jQuery to current document in Magento 2 When template load by ajaxRedirect page using plugin in Magento 2Magento 2 - Update quantity and totals of cart page without page reload?Magento 2: Quote data not loaded on knockout checkoutMagento 2 : I need to change add to cart success message after adding product into cart through pluginMagento 2.2.5 How to add additional products to cart from new checkout step?Magento 2 Add error/success message with knockoutCan't validate Post Code on checkout page

                                Fil:Tokke komm.svg

                                Where did Arya get these scars? Unicorn Meta Zoo #1: Why another podcast? Announcing the arrival of Valued Associate #679: Cesar Manara Favourite questions and answers from the 1st quarter of 2019Why did Arya refuse to end it?Has the pronunciation of Arya Stark's name changed?Has Arya forgiven people?Why did Arya Stark lose her vision?Why can Arya still use the faces?Has the Narrow Sea become narrower?Does Arya Stark know how to make poisons outside of the House of Black and White?Why did Nymeria leave Arya?Why did Arya not kill the Lannister soldiers she encountered in the Riverlands?What is the current canonical age of Sansa, Bran and Arya Stark?